For Angular Momentum Operator L, prove [Lx,Ly] = ihLz

In summary, the homework statement is that for an angular momentum operator ~L =ˆiLx +ˆjˆLy + ˆkˆLz = ˆr × ˆp, prove that [ˆLx, ˆLy] = i\hbarˆLz, [ˆLx, ˆLz] = −i\hbarˆLy, [ˆL2, ˆLx] = 0, [p^{2}, ˆLx] = 0, [r^{2}, ˆLx] = 0, [ˆLx, ˆy
  • #1
Diomarte
26
0

Homework Statement



For an angular momentum operator ~L =ˆiLx +ˆjˆLy + ˆkˆLz = ˆr × ˆp, prove that
[ˆLx, ˆLy] = i[itex]\hbar[/itex]ˆLz,
[ˆLx, ˆLz] = −i[itex]\hbar[/itex]ˆLy,
[ˆL2, ˆLx] = 0,
[[itex]p^{2}[/itex], ˆLx] = 0,
[[itex]r^{2}[/itex], ˆLx] = 0,
[ˆLx, ˆy] = i[itex]\hbar[/itex]ˆz,
[ˆLx, ˆpy] = i[itex]\hbar[/itex]ˆPz.

**Note: I'm really only looking for help for the first part here, but any suggestions on how to approach the others would be appreciated. I figure I can't approach those, until I understand fully how to work through the first one.**

Homework Equations



Px = −i[itex]\hbar[/itex] d/dx
Py = −i[itex]\hbar[/itex] d/dy
Pz = −i[itex]\hbar[/itex] d/dz

Lx = yPz - zPy
Ly = zPx - xPz
Lz = xPy - yPx

The Attempt at a Solution



I'm simply looking for help on [Lx,Ly] = −i[itex]\hbar[/itex] Lz right now:

Take [Lx,Ly] = [yPz - zPy, zPx - xPz] = [yPz, zPx]-[yPz, xPz]-[zPy, zPx]+[zPy, xPz]

Looking at the first term:
[yPz,zPx] = (yPz zPx - zPx yPz)
==> first term here: yPz zPx = y(-i[itex]\hbar[/itex])d/dz z(-i[itex]\hbar[/itex])d/dx
==> second term here: zPx yPz = z(-i[itex]\hbar[/itex])d/dx y(-i[itex]\hbar[/itex])d/dz

==> first term simplified: -[itex]\hbar^{2}[/itex]yz d/dz d/dx ?
==> second term simplified: -[itex]\hbar^{2}[/itex]zy d/dx d/dz ?

Subtracting the second term from the first term, I get:
[ -[itex]\hbar^{2}[/itex]yz d/dz d/dx + [itex]\hbar^{2}[/itex]zy d/dx d/dz ] ?

The "?" is to represent where I'm having trouble with the algebra and differentiation.
I'm not sure that these operations actually work out like this. Also, the next step I have in my notes here says:

[yPz,zPx] = -[itex]\hbar^{2}[/itex]yz [itex]\partial^{2}[/itex]/dzdx - [itex]\hbar^{2}[/itex]y [itex]\partial[/itex]/dx + [itex]\hbar^{2}[/itex]yz [itex]\partial^{2}[/itex]/dxdz = -[itex]\hbar^{2}[/itex]y [itex]\partial[/itex]/dx

It could be a simple chain rule issue that I'm just not seeing, but would somebody please explain to me how you obtain each of these three terms? The first and third terms I seem to have no problem coming up with, but the second term is throwing me, especially since there isn't a matching term to follow before the [itex]\hbar^{2}[/itex]yz [itex]\partial^{2}[/itex]/dxdz term. I think the second term is actually more common sense than the first and third terms since d/dz z = 1... As I said, anyone who can explain how to arrive at each of these terms, your assistance would be greatly appreciated.
 
Last edited:
Physics news on Phys.org
  • #2
Don't bother with actual representation in space (that momentum become a partial derivative operator).

Since [itex]L_z = x p_y - y p_x[/itex] etc, just write out the commutator in Cartesian coordinates:
[itex][L_x, L_y] = [y p_z - z p_y, z p_x - x p_z][/itex] and use properties of the commutator to get rid/factor out anything that commutes.

You should end up with [itex][L_x, L_y] = y p_x [p_z, z] + x p_y [z, p_z][/itex]. Then apply the canonical commutation relation.
 
Last edited:
  • #3
You're not evaluating the products correctly. You need to apply the product rule, not the chain rule.
[tex]y\frac{\partial}{\partial z} z\frac{\partial}{\partial x}
= y \left(\frac{\partial}{\partial z} z\right) \frac{\partial}{\partial x} + y z \left(\frac{\partial}{\partial z}\frac{\partial}{\partial x}\right) = y\frac{\partial}{\partial x} + \frac{\partial^2}{\partial z\partial x}[/tex]
But as mathfeel implied, this is the long, tedious way of doing it. I suggest you follow his suggestion.
 
  • #4
Thank you both. I was able to solve that problem, I have moved forward in this problem and am stuck on the [r^2, Lx] portion. I know that the x[x,Lx] + [x,Lx]x = 0 and that I'm left with the y and z terms, but I'm having trouble with those. Do you have any suggestions towards that approach?

I've got [y,Lx] = [y, xPz-zPy] ==> [y,yPz] - [y, zPy]
and following [A,BC] = [A,B]C + B[A,C] this expands out to be

[y,y]Pz + y[y,Pz] - [y, z]Py + z[y,Py]

If I understand this correctly, the [y,y] term goes to zero, as it must, and so must [y,z]
so I'm left with two terms y[y,Pz] - z[y,Py]

Is this correct?
 
  • #5
That's correct, but [y, pz] also vanishes.
 
  • #6
so I'm left only with -z[y,Py] huh?

I've worked it out a few times, and if I remember correctly [y,Pz] clearly vanishes when operating on some form of dummy function, but without actually working it out, can you explain briefly why exactly it vanishes?
 
  • #7
"The only operator in Lz that does not commute with x, is Px..." but why?
 
  • #8
Because when you take the partial with respect to z, you hold y constant.
 
  • #9
vela said:
Because when you take the partial with respect to z, you hold y constant.

Most definitely, but forgive my lack of understanding at 1:00am , but is there a physical explanation, or is this just one of those things that is only mathematical?
 
  • #10
Never mind, Vela, you're amazing and awesome, and I really appreciate all the help you've given me. You answer just about every question I ask, and for that I thank you. I totally just woke up, and now things are making sense again. I think I need a short break.
 
  • #11
I do have one last question for you, does [Px, Py] commute? (Do each part of [Pi,Pj] commute?
 
  • #12
P_x and P_y commute, of course. This is postulated in a tensorial notation [p_i,p_j] = 0, [itex]\forall[/itex] [itex] i,j=\bar{1,3} [/itex]. When i=1. j=2, it means exactly [p_x,p_y]=0.
 
  • #13
Diomarte said:
I do have one last question for you, does [Px, Py] commute? (Do each part of [Pi,Pj] commute?

If you are in doubt, think intuitively back to differential operator. Can you change the order of taking partial derivative in the x and in the y direction?
 

1. What is the meaning of [Lx, Ly]?

The notation [Lx, Ly] represents the commutator of the operators Lx and Ly. This is a mathematical operation that measures how the two operators interact with each other.

2. How is the commutator of two operators defined?

The commutator of two operators A and B is defined as [A, B] = AB - BA. In other words, it is the difference between the product of the two operators and the product in reverse order.

3. How does the commutator relate to the angular momentum operator?

The commutator [Lx, Ly] is used to measure the interaction between the operators Lx and Ly, which are components of the angular momentum operator L. This commutator is important in understanding the behavior of angular momentum in quantum mechanics.

4. What is the significance of proving [Lx, Ly] = ihLz?

Proving this commutation relation is important because it demonstrates the consistency of the mathematical framework used in quantum mechanics. It also helps us understand the behavior of angular momentum in systems described by quantum mechanics.

5. What does the constant 'ih' represent in the commutation relation?

The constant 'ih' represents the reduced Planck's constant, also known as Dirac's constant. It has a value of approximately 1.0545718 × 10^-34 joule-seconds in SI units. This constant is important in quantum mechanics as it relates the energy of a particle to its frequency and wavelength.

Similar threads

  • Advanced Physics Homework Help
Replies
6
Views
1K
  • Advanced Physics Homework Help
Replies
10
Views
579
Replies
27
Views
2K
  • Advanced Physics Homework Help
Replies
10
Views
2K
  • Advanced Physics Homework Help
Replies
2
Views
2K
Replies
3
Views
402
  • Advanced Physics Homework Help
Replies
1
Views
4K
  • Advanced Physics Homework Help
Replies
3
Views
687
  • Advanced Physics Homework Help
Replies
1
Views
1K
  • Advanced Physics Homework Help
Replies
4
Views
1K
Back
Top